Manager: Our company's mail–order sales have recently increased 25 percent. This increase started around the time we ...

letc on December 9, 2018

Test 66, section 4, Q12

Can someone explain why the answer is A? It seems to me that it weakens it. My initial answer was C.

Replies
Create a free account to read and take part in forum discussions.

Already have an account? log in

Ravi on December 13, 2018

Hi @letc, excellent question. The manager's argument is a classic case of taking a correlation/coincidental factors and extrapolating a causal relationship from the correlation. In order to strengthen the manager's argument, we can do a a few different things, including blocking alternative causes or hypotheses.

In reading answer choice (A), we have a fact that would provide corroborating evidence for the manager's argument. If (A) is true, then it blocks an alternative explanation of the economy simply being better all around.

If (A) weren't true, then who is to say that the manager's company's 25 percent increase in mail-order sales wasn't the result of greater economic activity across all industries? (A) blocks that potential cause/hypothesis and strengthens the correlation between the company's mail-order sales rising and the company offering unlimited free shipping.

Answer choice (C) does nothing to strengthen the argument because we are not concerned with the company's profits from mail-order sales, we are concerned with the increase in sales and are seeking to strengthen the manager's explanation for the increase in sales (offering unlimited shipping).

Additionally, if (C) were true, it does not block the alternative hypothesis of there simply being greater economic activity and that being the primary reason for the company's increase in mail-order sales. Let's say (C) is true. So what? Maybe the economy is just stronger all around. Whether or not the company's profits from mail-order sales have increased has nothing to do with the argument.

Hopefully this helps. Let us know if you have any more questions—we're here to help!

letc on December 13, 2018

This makes sense now, A is the one that MOST strengthens and eliminates all possibilities. Thank you!!

Ravi on December 14, 2018

Happy to help, @letc!